Jump to content

Photo

$\frac{a}{b^3}+\frac{b}{c^3}+\frac{c}{a^3} \geq 1$


  • Please log in to reply
12 replies to this topic

#1
caybutbixanh

caybutbixanh

    Trung úy

  • Thành viên
  • 888 posts

Bài 1: Cho x,y,z dương thỏa mãn $xyz=1.$ Tìm giá trị nhỏ nhất của :

a,$P=x+y+z;$

b,$P=\sqrt{x}+\sqrt{y}+\sqrt{z}$

c,$P=\frac{1}{\sqrt{x}}+\frac{1}{\sqrt{y}}+\frac{1}{\sqrt{z}};$

Bài 2: Cho a,b,c dương thỏa mãn abc=1.CMR:

a,$a^3+b^3+c^3 \geq a+b+c$

b,$a^3+b^3+c^3 \geq a^2+b^2+c^2$

c,$(a+\frac{1}{b}-1).(b+\frac{1}{c}-1).(c+\frac{1}{a}-1) \leq 1$

Bài 3 : Cho a,b,c là độ dài ba cạnh của 1 tam giác. CMR:

$\frac{a}{a+b-c}+\frac{b}{b+c-a}+\frac{c}{a+c-b} \geq 3$

Bài 4: Cho a,b,c dương thỏa mãn $a+b+c=abc.$CMR:

$\frac{a}{b^3}+\frac{b}{c^3}+\frac{c}{a^3} \geq 1$


KẺ MẠNH CHƯA CHẮC ĐÃ THẮNG



MÀ KẺ THẮNG MỚI CHÍNH LÀ KẺ MẠNH!.



(FRANZ BECKEN BAUER)




ÔN THI MÔN HÓA HỌC TẠI ĐÂY.


#2
HoangVienDuy

HoangVienDuy

    Sĩ quan

  • Thành viên
  • 309 posts

Bài 3 : Cho a,b,c là độ dài ba cạnh của 1 tam giác. CMR:

$\frac{a}{a+b-c}+\frac{b}{b+c-a}+\frac{c}{a+c-b} \geq 3$

$a+b-c=x;b+c-a=y;a+c-b=z$

bài toán viết thành:

$\frac{x+z}{2x}+\frac{x+y}{2y}+\frac{y+z}{2z}=\frac{1}{2}.\left ( 3+\frac{z}{x}+\frac{x}{y}+\frac{y}{z} \right )\geq \frac{1}{2}.(3+3)=3$


Edited by Super Fields, 20-05-2015 - 21:58.

Có một người đi qua hoa cúc

Có hai người đi qua hoa cúc

Bỏ lại sau lưng cả tuổi thơ mình...

FB:https://www.facebook.com/hoang.vienduy


#3
HoangVienDuy

HoangVienDuy

    Sĩ quan

  • Thành viên
  • 309 posts

Bài 4: Cho a,b,c dương thỏa mãn $a+b+c=abc.$CMR:

$\frac{a}{b^3}+\frac{b}{c^3}+\frac{c}{a^3} \geq 1$

đặt $\frac{1}{a}=x;\frac{1}{b}=y;\frac{1}{c}=z$

nên :1=xy+yz+zx

Bài toán viết thành: $\sum \frac{y^{3}}{x}=\sum \frac{y^{4}}{xy}\geq \frac{(\sum x^{2})^{2}}{\sum xy}\geq \sum x^{2}\geq \sum xy=1 $ (đpcm)


Edited by HoangVienDuy, 20-05-2015 - 22:04.

Có một người đi qua hoa cúc

Có hai người đi qua hoa cúc

Bỏ lại sau lưng cả tuổi thơ mình...

FB:https://www.facebook.com/hoang.vienduy


#4
Hoangphan

Hoangphan

    Lính mới

  • Thành viên
  • 5 posts

 

Bài 4: Cho a,b,c dương thỏa mãn $a+b+c=abc.$CMR:

$\frac{a}{b^3}+\frac{b}{c^3}+\frac{c}{a^3} \geq 1$

$GT\leftrightarrow \frac{1}{ab}+\frac{1}{bc}+\frac{1}{ca}=1$

Ta có:

 $\frac{a}{b^3}+\frac{1}{a^2}+\frac{1}{a^2}\geq \frac{3}{ab}$

Tương tự, cộng lại ta được:

$\frac{a}{b^3}+\frac{b}{c^3}+\frac{c}{a^3}+2(\frac{1}{a^2}+\frac{1}{b^2}+\frac{1}{c^2})\geq 3(\frac{1}{ab}+\frac{1}{bc}+\frac{1}{ca})$

Vì $\frac{1}{a^2}+\frac{1}{b^2}+\frac{1}{c^2}\geq \frac{1}{ab}+\frac{1}{bc}+\frac{1}{ca} => đpcm$



#5
hoctrocuaHolmes

hoctrocuaHolmes

    Thượng úy

  • Thành viên
  • 1013 posts

Bài 1: Cho x,y,z dương thỏa mãn $xyz=1.$ Tìm giá trị nhỏ nhất của :

a,$P=x+y+z;$

b,$P=\sqrt{x}+\sqrt{y}+\sqrt{z}$

c,$P=\frac{1}{\sqrt{x}}+\frac{1}{\sqrt{y}}+\frac{1}{\sqrt{z}};$

Bài 2: Cho a,b,c dương thỏa mãn abc=1.CMR:

a,$a^3+b^3+c^3 \geq a+b+c$

b,$a^3+b^3+c^3 \geq a^2+b^2+c^2$

c,$(a+\frac{1}{b}-1).(b+\frac{1}{c}-1).(c+\frac{1}{a}-1) \leq 1$

1.Áp dụng bất đẳng thức AM-GM cho 3 số dương ta có

a)$x+y+z\geq \sqrt[3]{xyz}=3 (xyz=1)$

Dấu ''='' xảy ra $\Leftrightarrow x=y=z=1$

b)$\sqrt{x}+\sqrt{y}+\sqrt{z}\geq 3\sqrt[3]{\sqrt{xyz}}=3$ (vì $xyz=1$)

Dấu ''='' xảy ra $\Leftrightarrow x=y=z=1$

c)$\frac{1}{\sqrt{x}}+\frac{1}{\sqrt{y}}+\frac{1}{\sqrt{z}}\geq 3\sqrt[3]{\frac{1}{\sqrt{xyz}}}=3$ (vì $xyz=1$)

Dấu ''='' xảy ra $\Leftrightarrow x=y=z=1$



#6
tonarinototoro

tonarinototoro

    Trung sĩ

  • Thành viên
  • 174 posts

$GT\leftrightarrow \frac{1}{ab}+\frac{1}{bc}+\frac{1}{ca}=1$

Ta có:

 $\frac{a}{b^3}+\frac{1}{a^2}+\frac{1}{a^2}\geq \frac{3}{ab}$

Tương tự, cộng lại ta được:

$\frac{a}{b^3}+\frac{b}{c^3}+\frac{c}{a^3}+2(\frac{1}{a^2}+\frac{1}{b^2}+\frac{1}{c^2})\geq 3(\frac{1}{ab}+\frac{1}{bc}+\frac{1}{ca})$

Vì $\frac{1}{a^2}+\frac{1}{b^2}+\frac{1}{c^2}\geq \frac{1}{ab}+\frac{1}{bc}+\frac{1}{ca} => đpcm$

ngược dấu!



#7
tonarinototoro

tonarinototoro

    Trung sĩ

  • Thành viên
  • 174 posts

 

Bài 3 : Cho a,b,c là độ dài ba cạnh của 1 tam giác. CMR:

$\frac{a}{a+b-c}+\frac{b}{b+c-a}+\frac{c}{a+c-b} \geq 3$

 

cách khác

do $a,b,c$ là 3 cạnh tam giác nên $a+b-c,b+c-a,c+a-b> 0$

áp dụng AM-GM có $\sum \frac{a}{a+b-c}\geq 3\sqrt[3]{\frac{abc}{\left ( a+b-c \right )\left ( b+c-a \right )\left ( c+a-b \right )}}$

chỉ cần cm $abc\geq \coprod \left ( a+b-c \right )$ => cái này luôn đúng theo schur

hoặc cm = AM-GM có $\sqrt{\left ( a+b-c \right )\left ( b+c-a \right )}\leq \frac{a+b-c+b+c-a}{2}=\frac{2b}{2}=b$ thiết lập các bđt tương tự rồi nhân lại có đpcm


Edited by tonarinototoro, 20-05-2015 - 22:39.


#8
tonarinototoro

tonarinototoro

    Trung sĩ

  • Thành viên
  • 174 posts

Bài 4: Cho a,b,c dương thỏa mãn $a+b+c=abc.$CMR:

$\frac{a}{b^3}+\frac{b}{c^3}+\frac{c}{a^3} \geq 1$

gt$\Rightarrow \frac{1}{ab}+\frac{1}{bc}+\frac{1}{ca}= 1$

$\frac{a}{b^{3}}+\frac{b}{c^{3}}+\frac{1}{ab}\geq \frac{3}{bc}$

thiết lập các bđt tương tự rồi cộng lại có $2\sum \frac{a}{b^{3}}+\sum \frac{1}{ab}\geq 3\sum \frac{1}{ab}\Rightarrow 2\sum \frac{a}{b^{3}}\geq 2\sum \frac{1}{ab}=2\Rightarrow$đpcm



#9
congdaoduy9a

congdaoduy9a

    Sĩ quan

  • Thành viên
  • 338 posts

Bài 5 : Sẵn e xin post 1 câu này 

Cho các số thực thỏa mãn abc=1.CMR:

$\sum \frac{a^{3}}{(1+b)(1+c)}\geq \frac{3}{4}$



#10
nguyenhongsonk612

nguyenhongsonk612

    Thượng úy

  • Thành viên
  • 1451 posts

 

Bài 2: Cho a,b,c dương thỏa mãn abc=1.CMR:

a,$a^3+b^3+c^3 \geq a+b+c$

b,$a^3+b^3+c^3 \geq a^2+b^2+c^2$

c,$(a+\frac{1}{b}-1).(b+\frac{1}{c}-1).(c+\frac{1}{a}-1) \leq 1$

 

$a)$ $AM-GM$: $a+b+c\geq 3$; $a^3+1+1\geq 3a$

$\Rightarrow VT\geq 3(a+b+c)-6\geq a+b+c\Leftrightarrow a+b+c\geq 3$ (đúng)

Dấu "=" xảy ra khi $a=b=c=1$

$b)$ $AM-GM$: $a^3+a^3+1\geq 3a^2$

$\Rightarrow VT\geq 3(a^2+b^2+c^2)-3\geq 2(a^2+b^2+c^2)$ (đúng do $a^2+b^2+c^2\geq 3$)

$c)$ Đặt $a=\frac{x}{y};b=\frac{y}{z};c=\frac{z}{x}$ 

BĐT $\Leftrightarrow (x+y-z)(y+z-x)(z+x-y)\leq xyz$

Đến đây có thể khai triển ra dùng Schur bậc $3$ nhưng mình sẽ đi theo hướng khác

Đặt $x+y-z=A;y+z-x=B;z+x-y=C$

Ta xét các TH sau 

+)TH $1$: Cả $3$ số $A, B, C$ đều âm $\Rightarrow ABC< 0\leq xyz$

+) TH $2$: Trong $A, B, C$ có ít nhất một số âm

Không mất tính tổng quát giả sử $A< 0\Leftrightarrow x+y< z$

Ta có $B=y+z-x> 2y> 0;C=z+x-y> 2x> 0$ $\Rightarrow ABC< 0\leq xyz$

+) TH $3$: Cả ba số $A, B, C$ đều dương

Đến đây ghép cặp rồi sử dụng $AM-GM$. BĐT này đã quen thuộc 

Dấu "=" xảy ra khi và chỉ khi $a=b=c$


"...Từ ngay ngày hôm nay tôi sẽ chăm chỉ học hành như Stardi, với đôi tay nắm chặt và hàm răng nghiến lại đầy quyết tâm. Tôi sẽ nỗ lực với toàn bộ trái tim và sức mạnh để hạ gục cơn buồn ngủ vào mỗi tối và thức dậy sớm vào mỗi sáng. Tôi sẽ vắt óc ra mà học và không nhân nhượng với sự lười biếng. Tôi có thể học đến phát bệnh miễn là thoát khỏi cuộc sống nhàm chán khiến mọi người và cả chính tôi mệt mỏi như thế này. Dũng cảm lên! Hãy bắt tay vào công việc với tất cả trái tim và khối óc. Làm việc để lấy lại niềm vui, lấy lại nụ cười trên môi thầy giáo và cái hôn chúc phúc của bố tôi. " (Trích "Những tấm lòng cao cả")

~O) 


#11
caybutbixanh

caybutbixanh

    Trung úy

  • Thành viên
  • 888 posts

1.Áp dụng bất đẳng thức AM-GM cho 3 số dương ta có

a)$x+y+z\geq \sqrt[3]{xyz}=3 (xyz=1)$

Dấu ''='' xảy ra $\Leftrightarrow x=y=z=1$

b)$\sqrt{x}+\sqrt{y}+\sqrt{z}\geq 3\sqrt[3]{\sqrt{xyz}}=3$ (vì $xyz=1$)

Dấu ''='' xảy ra $\Leftrightarrow x=y=z=1$

c)$\frac{1}{\sqrt{x}}+\frac{1}{\sqrt{y}}+\frac{1}{\sqrt{z}}\geq 3\sqrt[3]{\frac{1}{\sqrt{xyz}}}=3$ (vì $xyz=1$)

Dấu ''='' xảy ra $\Leftrightarrow x=y=z=1$

Thực sự thì ban đâu mình cũng nghĩ như vậy....nhưng sau lại thấy không ổn vì chẳng lẽ...dễ quá vậy...mấy bài này mình lấy trong chuyên đề BDT luyện thi đại học của thầy  Lê Xuân ĐẠi......


KẺ MẠNH CHƯA CHẮC ĐÃ THẮNG



MÀ KẺ THẮNG MỚI CHÍNH LÀ KẺ MẠNH!.



(FRANZ BECKEN BAUER)




ÔN THI MÔN HÓA HỌC TẠI ĐÂY.


#12
leduylinh1998

leduylinh1998

    Thượng sĩ

  • Thành viên
  • 288 posts

Bài 4: Cho a,b,c dương thỏa mãn $a+b+c=abc.$CMR:

$\frac{a}{b^3}+\frac{b}{c^3}+\frac{c}{a^3} \geq 1$

$(\sum \frac{a}{b^3})(\sum \frac{1}{ab})\geq \left ( \sum \frac{1}{b^2} \right )^2\geq \left (\sum  \frac{1}{ab} \right )^2$

$\Rightarrow \sum \frac{a}{b^3}\geq \sum \frac{1}{ab}=1$



#13
tonarinototoro

tonarinototoro

    Trung sĩ

  • Thành viên
  • 174 posts

Bài 5 : Sẵn e xin post 1 câu này 

Cho các số thực thỏa mãn abc=1.CMR:

$\sum \frac{a^{3}}{(1+b)(1+c)}\geq \frac{3}{4}$

 AM-GM $a+b+c\geq 3\sqrt[3]{abc}=3$

lại AM-GM có $\frac{a^{3}}{\left ( b+1 \right )\left ( c+1 \right )}+\frac{b+1}{8}+\frac{c+1}{8}\geq \frac{3}{4}a$

thiết lập thêm 2 bđt tương tự rồi cộng lại có $\sum \frac{a^{3}}{\left ( b+1 \right )\left ( c+1 \right )}+\frac{1}{4}\sum a+\frac{3}{4} \geq \frac{3}{4}\sum a\Rightarrow \sum \frac{a^{3}}{\left ( b+1 \right )\left ( c+1 \right )}\geq \frac{1}{2}\sum a-\frac{3}{4}\geq \frac{1}{2}.3-\frac{3}{4}=\frac{3}{4}$

đẳng thức xảy ra khi $"a=b=c=1"$






1 user(s) are reading this topic

0 members, 1 guests, 0 anonymous users